What conditions are needed to get a stable limit cycle here?

In summary, the system given does not have a limit cycle and instead exhibits a Volterra-Lotka behavior with concentric cycles. A Hopf bifurcation could occur if and only if AC = 0, but this is not the case for the given system. Instead, the fixed point at (C/D, A/B) is a center and varying B or D will only change its location, not its type.
  • #1
JuanC97
48
0

Homework Statement


I want to find conditions over A,B,C,D to observe a stable limit cycle in the following system:
[tex] \frac{dx}{dt} = x \; (A-B y) \hspace{1cm} \frac{dy}{dt} = -y \; (C-D x) [/tex]

Homework Equations


Setting dx/dt=0 and dy/dt=0 you can find that (C/D , A/B) is a fixpoint.

The Attempt at a Solution


When I evaluate the Jacobian matrix in that point...
[tex]
J(x,y) = \left[
\begin{matrix}
A-B y & -B x
\\
D y & D x - C
\end{matrix} \right]
\: \Rightarrow \: J\left(\frac{C}{D}, \frac{A}{B}\right) =
\left[ \begin{matrix}
0 & -\frac{BC}{D}
\\
\frac{DA}{B} & 0
\end{matrix} \right]
[/tex]
Since λ1 = λ2 = 0, a Hopf bifurcation could occur (If and only if Tr(J)=0 ∧ Det(J)>0). As a consequence... it's necesary to have A,B,C,D such that AC > 0, however, by definition A,B,C,D are positive.

Does it mean that I will always have a bifurcation? If so, why am I getting stable and unstable orbits when I change the parameters?. Also, if this approach is not useful to determine the requirements to have a stable limit cycle, then, what could be the best approach? (Sorry if these questions sound kinda basic - I'm trying to learn about this topic by myself)
 
  • #3
This does not look like a system with a limit cycle, it looks like a Volterra-Lotka system, where you just have an infinite number of concentric cycles.
 
  • #4
epenguin said:
it looks like a Volterra-Lotka system
You're right, however I'll have to search a lot more about this topic. If you know any book that would be helpful, I'd appreciate it.

By the way, I said I was "getting stable orbits" but... well, I wasn't studying the system for a sufficient long period of time so, yeah, it's just like you already said it... concentric cycles but nothing like a stable orbit. Thanks for the comment :thumbup:
 
  • #5
At the moment I have forgotten stuff I knew even not that long ago, but I think with a small modification of this Volterra-Lotka system you can get limit cycles.

Others can suggest books, but you probably want fairly introductory or elementary. If your orientation is engineering, there is "nonlinear ordinary differential equations" by Jordan and Smith. Instead for biology, the most elementary if you can find it and written for biologists with little maths background is "modelling dynamic phenomena in molecular and cellular biology" by L A Segel. (1984). To read these books is to realize you could have worked most of it out yourself. Segel afterwards produced a number of books I have not seen, but which seem to be similar but taking it further with titles like "A primer on mathematical models in biology"
 
  • Like
Likes JuanC97
  • #6
JuanC97 said:

Homework Statement


I want to find conditions over A,B,C,D to observe a stable limit cycle in the following system:
[tex] \frac{dx}{dt} = x \; (A-B y) \hspace{1cm} \frac{dy}{dt} = -y \; (C-D x) [/tex]

Homework Equations


Setting dx/dt=0 and dy/dt=0 you can find that (C/D , A/B) is a fixpoint.

The Attempt at a Solution


When I evaluate the Jacobian matrix in that point...
[tex]
J(x,y) = \left[
\begin{matrix}
A-B y & -B x
\\
D y & D x - C
\end{matrix} \right]
\: \Rightarrow \: J\left(\frac{C}{D}, \frac{A}{B}\right) =
\left[ \begin{matrix}
0 & -\frac{BC}{D}
\\
\frac{DA}{B} & 0
\end{matrix} \right]
[/tex]
Since λ1 = λ2 = 0, a Hopf bifurcation could occur (If and only if Tr(J)=0 ∧ Det(J)>0). As a consequence... it's necesary to have A,B,C,D such that AC > 0, however, by definition A,B,C,D are positive.

Does it mean that I will always have a bifurcation? If so, why am I getting stable and unstable orbits when I change the parameters?. Also, if this approach is not useful to determine the requirements to have a stable limit cycle, then, what could be the best approach? (Sorry if these questions sound kinda basic - I'm trying to learn about this topic by myself)

The only bifurcations occur where [itex]AC = 0[/itex].

What happens when [itex]AC = 0[/itex] is that the fixed point at [itex](C/D, A/B)[/itex] merges with the trivial fixed point at [itex](0,0)[/itex]. This is not a Hopf bifurcation; it's something more complicated. When [itex]AC = 0[/itex] you end up with one or both of the coordinate axes consisting of degenerate fixed points.

A Hopf bifurcation occurs where a pair of complex conjugate eigenvalues crosses the imaginary axis, so that the real part changes sign. That's not what's happening here; it's just not possible to vary the parameters to make the real parts of the eigenvalues of the Jacobian at [itex](C/D, A/B)[/itex] anything other than zero. For all [itex]AC > 0[/itex] this fixed point is a center; varying [itex]B[/itex] or [itex]D[/itex] just changes the location of the fixed point, not its type.
 
  • Like
Likes JuanC97

1. What is a limit cycle?

A limit cycle is a type of behavior that occurs in dynamic systems, where the system's state variables exhibit a repeated pattern over time. In other words, the system's state variables will fluctuate between certain values in a continuous cycle.

2. What factors contribute to a stable limit cycle?

There are a few key conditions that must be met in order for a system to exhibit a stable limit cycle. These include having a non-linear system, having at least one state variable that is in a feedback loop, and having a positive feedback mechanism that reinforces the cycle.

3. How can you determine if a system will have a stable limit cycle?

One way to determine if a system will have a stable limit cycle is by analyzing its state variables and their relationships. If the system is non-linear and exhibits positive feedback, it is likely to have a stable limit cycle. Additionally, conducting simulations or experiments can also help to determine if a system will have a stable limit cycle.

4. What are the implications of a stable limit cycle in a system?

A stable limit cycle can have various implications depending on the specific system. In some cases, it can lead to predictable and consistent behavior, making it useful for controlling and regulating systems. However, in other cases, it can cause unwanted oscillations or instability in the system.

5. Can a stable limit cycle be disrupted or broken?

Yes, a stable limit cycle can be disrupted or broken if the conditions that support it are changed. For example, if the positive feedback mechanism is removed or the system becomes linear, the stable limit cycle may no longer occur. Additionally, external factors such as disturbances or noise can also disrupt a stable limit cycle.

Similar threads

  • Calculus and Beyond Homework Help
Replies
4
Views
608
  • Calculus and Beyond Homework Help
Replies
3
Views
287
  • Calculus and Beyond Homework Help
Replies
2
Views
164
  • Calculus and Beyond Homework Help
Replies
3
Views
342
  • Calculus and Beyond Homework Help
Replies
2
Views
837
  • Calculus and Beyond Homework Help
Replies
12
Views
995
  • Calculus and Beyond Homework Help
Replies
20
Views
467
  • Calculus and Beyond Homework Help
Replies
21
Views
1K
  • Calculus and Beyond Homework Help
Replies
3
Views
575
  • Calculus and Beyond Homework Help
Replies
6
Views
766
Back
Top